[obm-l] Re: [obm-l] Re: [obm-l] Teoria dos números

2018-03-29 Por tôpico Pedro José
ra chegar na fórmula, não consegui. Saudações, PJMS. Em 29 de mar de 2018 22:30, "Pedro José" <petroc...@gmail.com> escreveu: > Desculpe- me, não são divisores. São os únicos números que não são > co-primos de p^k. > > Em 29 de mar de 2018 22:25, "Pedro José&qu

[obm-l] Re: [obm-l] Re: [obm-l] Teoria dos números

2018-03-29 Por tôpico Pedro José
Desculpe- me, não são divisores. São os únicos números que não são co-primos de p^k. Em 29 de mar de 2018 22:25, "Pedro José" <petroc...@gmail.com> escreveu: > Boa noite! > Israel, > você é detalhista. > É fácil ver que se n = p^k, só haverá p^(k-1) divisores de p^k.

[obm-l] Re: [obm-l] Re: [obm-l] Teoria dos números

2018-03-29 Por tôpico Pedro José
PJMS.u Em 29 de mar de 2018 21:48, "Pedro José" <petroc...@gmail.com> escreveu: > Boa noite! > Não tenho editor de símbolos. Portanto. > Fi(n)= n . Produtório de ( p-1)/ p, onde p é primo e p divide n. > > Em 28 de mar de 2018 22:19, "Anderson Torres" < &g

[obm-l] Re: [obm-l] Re: [obm-l] Teoria dos números

2018-03-29 Por tôpico Pedro José
Boa noite! Não tenho editor de símbolos. Portanto. Fi(n)= n . Produtório de ( p-1)/ p, onde p é primo e p divide n. Em 28 de mar de 2018 22:19, "Anderson Torres" escreveu: > Em 28 de março de 2018 21:24, Israel Meireles Chrisostomo >

[obm-l] Re: [obm-l] distância constante

2018-03-29 Por tôpico Pedro José
Boa noite! Corrigindo MF =NG= x e EM=FN=y e não: MF=EG= x e EM = FE = y. Saudações, PJMS Em 29 de março de 2018 19:06, Pedro José <petroc...@gmail.com> escreveu: > Boa noite! > > Faça o desenho conforme o problema. > > Projete o ponto E em AB e chame de M. Projete o ponto

[obm-l] Re: [obm-l] distância constante

2018-03-29 Por tôpico Pedro José
Boa noite! Faça o desenho conforme o problema. Projete o ponto E em AB e chame de M. Projete o ponto G em AB e chame de N. Os triângulos EMF e GFM (ALA) são congruentes. MF=EG= x e EM = FE = y. BM=k= x. tg30 NC = l = y tg30 k + x + y + l = a = (x+y). (1 + tg30) ==> x + y = a/(1 + tg30) ==>

Re: [obm-l] Teoria dos numeros

2018-03-28 Por tôpico Pedro José
Bom dia! Não deu para compreender. Para cada terno (k,j,w) terá apenas uma raiz em x ou nenhuma. Mas para todo natural existe pelo menos um terno que atenda a sua proposição. w=x ; k=1 e j=2. Saudações, PJMS Em 27 de março de 2018 22:28, Israel Meireles Chrisostomo <

[obm-l] Re: [obm-l] Re: [obm-l] Re: [obm-l] Re: [obm-l] Re: [obm-l] Teoria dos números

2018-03-26 Por tôpico Pedro José
omo k-2 deve ser inteiro positivo, k só pode ser 3 e, portanto: >>> > 2/(t-1) + 2/(u-1) + 1/((t-1)(u-1)) = 1 ==> >>> > (2 + 1/(t-1))/(u-1) = 1 - 2/(t-1) ==> >>> > u = 1 + (2t - 1)/(t - 3) = 3 + 5/(t-3) ==> >>> > t = 4 e u = 8 ou t = 8 e u = 4 (nã

[obm-l] Re: [obm-l] Re: [obm-l] Teoria dos números

2018-03-23 Por tôpico Pedro José
Achar s, t tais que (s-1)(t-1) | st - 1, com 1 < s < t. > > 2018-03-23 15:38 GMT-03:00 Pedro José <petroc...@gmail.com>: > >> Boa tarde! >> >> Aproveitando que deu o que falar o problema postado pelo Douglas, tem um >> que achei mais interessante. >>

[obm-l] Re: Teoria dos números

2018-03-23 Por tôpico Pedro José
Boa tarde! Não entendi, os e-mails que estou enviando estão caindo anexo a esse, mandei um propondo um problema e não o vi aparecer. Postei novamente. E os dois caíram aqui, embora fosse uma mensagem nova. Desculpem-me, PJMS Em 23 de março de 2018 15:38, Pedro José <petroc...@gmail.

[obm-l] Teoria dos números

2018-03-23 Por tôpico Pedro José
Boa tarde! Aproveitando que deu o que falar o problema postado pelo Douglas, tem um que achei mais interessante. (s-1)(t-1).(u-1) | stu -1, com s, t, u inteiros e 1

[obm-l] Re: [obm-l] Re: [obm-l] Re: [obm-l] Re: [obm-l] Teoria dos números

2018-03-23 Por tôpico Pedro José
qui deixou de ser conjectura. Foi provado, na grosseria, por substituição mas foi. Saudações, PJMS Em 23 de março de 2018 11:07, Pedro José <petroc...@gmail.com> escreveu: > Bom dia! > Anderson, > o Gugu já avançou, em uma nota acima. E é passível. > Revendo a solução d

[obm-l] Teoria dos números

2018-03-23 Por tôpico Pedro José
Boa tarde! Seguindo a linha do Douglas, tem um que acho bem legal. (s-1) (t-1)(u-1) | stu -1 s,t, u estritamente naturais e s

[obm-l] Re: [obm-l] Re: [obm-l] Re: [obm-l] Re: [obm-l] Teoria dos números

2018-03-23 Por tôpico Pedro José
Bom dia! Anderson, o Gugu já avançou, em uma nota acima. E é passível. Revendo a solução do Ralph, fica claro que essa transformação seria de valia. Pois essa transformação leva a : a = (y+z)/2 b= (x+z)/2 c= (x+y)/2 Então na ordem que o Ralph apresentou: 1/2*(2x+y+z)(x+2y+z)(x+y+2z)=1 (b+c) dá

[obm-l] Re: [obm-l] Re: [obm-l] Re: [obm-l] Re: [obm-l] Teoria dos números

2018-03-22 Por tôpico Pedro José
Boa noite! Nem havia reparado que a transformação do Gugu, foi feita em composição com a anterior que fora postada. Acabou sendo a mesma que postei. Escolhi porque fazia sumir os termos com expoente 3. Saudações, PJMS Em 22 de mar de 2018 22:59, "Pedro José" <petroc...@gmail.

[obm-l] Re: [obm-l] Re: [obm-l] Re: [obm-l] Re: [obm-l] Teoria dos números

2018-03-22 Por tôpico Pedro José
Boa noite! Vi duas proposições de substituições de variáveis, nas notas anteriores e ratifico os questionamentos do Cláudio. Aventurei uma substituição: a=x+y ; b=x+z; c = y + z. Aí, na munheca cancelam-se os termos com expoentes cúbicos. E separando os termos de (a+b)*(a+c), no que sobra, chega-

[obm-l] Re: [obm-l] Re: [obm-l] Re: [obm-l] Re: [obm-l] Re: [obm-l] Teoria dos números

2018-03-20 Por tôpico Pedro José
1,-1,2); (-1,2;-1); (2,-1,-1) Ralph, Fiz esse salseiro todo, ao invés de fatorar. E olha, que ontem estava orgulhoso de ter achado a solução. Saudações, PJMS Em 20 de março de 2018 12:10, Pedro José <petroc...@gmail.com> escreveu: > É acabou me ajudando. Resolvi de uma outra form

[obm-l] Re: [obm-l] Re: [obm-l] Re: [obm-l] Re: [obm-l] Re: [obm-l] Teoria dos números

2018-03-20 Por tôpico Pedro José
y+2z)=1 > (2x+y+z)(x+2y+z)(x+y+2z)=2 > > Confiram se eu nao errei contas... Mas agora ficou **bem** facil! :D > > Abraco, Ralph. > > 2018-03-19 14:33 GMT-03:00 Pedro José <petroc...@gmail.com>: > >> Bom dia! >> >> Estou só conjecturando. Pois, não c

[obm-l] Re: [obm-l] Re: [obm-l] Re: [obm-l] Teoria dos números

2018-03-19 Por tôpico Pedro José
solução da equação acima para inteiros. Em 19 de março de 2018 14:14, Claudio Buffara <claudio.buff...@gmail.com> escreveu: > Podem existir soluções não triviais envolvendo inteiros negativos. > > 2018-03-19 10:17 GMT-03:00 Pedro José <petroc...@gmail.com>: > >> Bom dia!

[obm-l] Re: [obm-l] Teoria dos números

2018-03-19 Por tôpico Pedro José
Bom dia! Poderia postar a solução? Não consegui achar nenhuma restrição para trabalhar num subconjunto pequeno dos inteiros. Creio que vá ser apenas a trivial (0,0,1) e suas permutações. grato, PJMS Em 13 de março de 2018 20:19, Douglas Oliveira de Lima < profdouglaso.del...@gmail.com>

Re: [obm-l] Probabilidade

2017-12-07 Por tôpico Pedro José
totais. [image: Imagem inline 3] p = k!. M / U Em 7 de dezembro de 2017 18:18, Lucas Reis <lucasvianar...@gmail.com> escreveu: > Acredito que mudar a primeira escolha seria permutar a primeira fazenda na > resolução Pedro José. Como são três opções com as mesmas

Re: [obm-l] Probabilidade

2017-12-07 Por tôpico Pedro José
Bom dia! Resolvendo por grafo. Para a primeira escolha sendo T há 90 ocorrências. Então o segundo com terceiro deverá ser BV e MV ou MV e BV, o que daria 2*90*110*80. Note que se mudar a primeira escolha, também dará 2*90*110*80. Então serão 6*90*110*80 chances favoráveis. O universo é

[obm-l] Re: [obm-l] Re: [obm-l] polinômios

2017-11-28 Por tôpico Pedro José
Bom dia! O Ralph seguiu o caminho certo. Contagem é para coisas distintas. Multiplicidade da raiz já é outro conceito. A solução do Ralph foi perfeita, pois, além de considerar as quatros raízes, não fez restrição à multiplicidade dessas raízes. Em 27 de novembro de 2017 21:51, Ralph Teixeira

[obm-l] Re: [obm-l] Re: [obm-l] Re: [obm-l] Teoria dos números

2017-11-23 Por tôpico Pedro José
aso particular 1/3^2002, seriam 955 algarismos zeros inseridos a esquerda. Saudações, PJMS Em 22 de novembro de 2017 11:45, Pedro José <petroc...@gmail.com> escreveu: > Bom dia! > > Não entendi como o Anderson chegou a solução, para determinar o período > propriamente dito. Todavia

[obm-l] Re: [obm-l] Re: [obm-l] Re: [obm-l] Teoria dos números

2017-11-22 Por tôpico Pedro José
Bom dia! Não entendi como o Anderson chegou a solução, para determinar o período propriamente dito. Todavia fiz um experimento e realmente dá certo para (10^k-1)/3^n. Acho que ele se enganou e reportou n somente ao invés de 3^n. Todavia, em alguns casos, precisa colocar algarismos zeros a

[obm-l] Re: [obm-l] Aritmética modular

2017-11-21 Por tôpico Pedro José
Boa noite! Não saiu a figura (https://en.wikipedia.org/wiki/Euler's_totient_function) caso não consiga visualizar e até por propósito, o certo teria sido citar a fonte da figura.: .[image: Imagem inline 1] onde p é primo e p divide n Em 21 de novembro de 2017 20:08, Pedro José <pet

[obm-l] Re: [obm-l] Aritmética modular

2017-11-21 Por tôpico Pedro José
Boa noite! a) (300,1001) = 1. 1001 = 7*11*13; então φ (1001) = 6*10*12 = 720. Para um caso geral, [image: {\displaystyle \varphi (n)=n\prod _{p\mid n}\left(1-{\frac {1}{p}}\right),}] onde p é primo e p divide n. 300^3000 = 300^ (4*720 + 120) = 300^120 mod 1001. Não adiantou nada, o resto 120

Re: [obm-l] Probabilidade

2017-11-17 Por tôpico Pedro José
leu Pedro também achei esquisito. > > Douglas Oliveira. > > Em 17 de nov de 2017 16:49, "Pedro José" <petroc...@gmail.com> escreveu: > >> Boa tarde! >> >> Não ficou claro o enunciado. Primeiramente cita que o lançamento é >> simultâneo, depois

Re: [obm-l] Probabilidade

2017-11-17 Por tôpico Pedro José
Boa tarde! Não ficou claro o enunciado. Primeiramente cita que o lançamento é simultâneo, depois que Alfredo é o primeiro a jogar. tem uma vírgula seguida da expressão não há vencedor que não faz o menor sentido... Supondo que os lançamentos são intercalados. E que se uma pessoa atinge a soma 10

Re: [obm-l]

2017-10-19 Por tôpico Pedro José
Boa tarde! Compreendido! Realmente, não há necessidade da restrição, quanto a serem medidas de lados de triângulos. Grato, PJMS. Em 19 de outubro de 2017 11:51, Israel Meireles Chrisostomo < israelmchrisost...@gmail.com> escreveu: > Pedro José, tudo bem! Talvez a condição de que a,

Re: [obm-l]

2017-10-19 Por tôpico Pedro José
Bom dia! Israel, desculpe-me, mais uma vez, fui precipitado, você só se referenciou a condição de existência do triângulo e não a restrição de abc=1. Saudações, PJMS Em 19 de outubro de 2017 10:18, Pedro José <petroc...@gmail.com> escreveu: > Bom dia! > > Desculpe-me, Israel. Mas

Re: [obm-l]

2017-10-19 Por tôpico Pedro José
Bom dia! Desculpe-me, Israel. Mas não compreendi o seu desenvolvimento. Mas tem algo de errado. E as restrições se fazem necessárias. Para a=4, b = 36 e c= 9, temos S = 7/18 e t = 11. Portanto, fura a hipótese de que S >=t. Saudações, PJMS Em 18 de outubro de 2017 22:18, Israel Meireles

Re: [obm-l] Fibonacci teoria dos numeros

2017-09-05 Por tôpico Pedro José
Boa tarde! O programa comera o F_28830 que é igual a zero. Desconsiderar o exposto anteriormente. Em 5 de setembro de 2017 16:25, Pedro José <petroc...@gmail.com> escreveu: > Boa tarde! > > Douglas, > > esse problema você viu em algum local ou foi uma conjectura sua?

Re: [obm-l] Fibonacci teoria dos numeros

2017-09-05 Por tôpico Pedro José
o, a hipótese é falsa. Deve ter um modo mais elegante para mostrar que a proposição é fasla. Sds, PJMS Em 5 de setembro de 2017 10:16, Pedro José <petroc...@gmail.com> escreveu: > Bom dia! > > Eu pensei que entendera, porém, os números não são sequenciais. > Se nós tivermo

Re: [obm-l] Fibonacci teoria dos numeros

2017-09-05 Por tôpico Pedro José
não daria, por exemplo, se consideramos A_1 = 3 e A_2 = 7 e Ai = A_i-1 + A_i-2, não haveria um número múltiplo de 8. A sequência mod 8 ficaria: 3, 7, 2, 1, 3, 4, 7, 3, 2, 5, 7, 4, 3, 7, 2, 1, 3, 4, 7, 3, 2, 5, 7, 4... Saudações, PJMS Em 4 de setembro de 2017 16:48, Pedro José <petroc...@gma

Re: [obm-l] Fibonacci teoria dos numeros

2017-09-04 Por tôpico Pedro José
Boa tarde! Nehab, não consegui entender o restante da solução, mas ele usou o sinal de igual para congruência por comodidade de edição, e até pela lei de formação da sequência, só o segundo e terceiro termos são iguais, quando se admite que comece de zero, ou os dois primeiros, para a corrente

Re: [obm-l] Passageiros em fila (probabilidade)

2017-09-01 Por tôpico Pedro José
Bom dia! Eu houvera entendido tão pouco, e como a princípio não entendi a premissa, me silenciei. Foi boa a sua pergunta. A mim, pareceu-me que o primeiro da fila sentará no primeiro assento e o segundo no segundo e assim por diante, até o louco quebrar ou não a cadeia. Só que o enunciado não faz

Re: [obm-l] Fibonacci teoria dos numeros

2017-09-01 Por tôpico Pedro José
Bom dia! Desculpem-me, mas fiz lambança, a fatoração pode ter primos repetidos, ou seja elevados a algum expoente diferente de 1. Destarte, a solução acima não atende. Tenho que se procurar mais. Em 31 de agosto de 2017 20:29, Pedro José <petroc...@gmail.com> escreveu: > Boa noite! >

Re: [obm-l] Fibonacci teoria dos numeros

2017-08-31 Por tôpico Pedro José
Boa noite! Desculpe-me, mas não entendi. Para usar a propriedade acima, teria que provar que o número natural w (no proposto pelo Douglas era n, mudei para não confundir) divide f_{(m,n)}, o que dá mesmo. Por exemplo se fizer m= 278 e n = 2085, (m,n) = 139 então f_139 = (f_278,f_2085). Todavia

[obm-l] Re: [obm-l] Re: [obm-l] Questão

2017-08-28 Por tôpico Pedro José
Bom dia! Daniel, eu já me sinto gratificado quando consigo resolver algo. Não sou matemático, sou um pitaqueiro, com alto grau curiosidade e matemática é uma das minhas curiosidades preferidas. O que mais me fascina, é que sou totalmente crente em que um modelo matemático formulado com estrutura,

[obm-l] Re: [obm-l] Divisão prolongada e série

2017-08-24 Por tôpico Pedro José
Boa tarde! a1, a2, a3 pertencente a {0, 1, 2, 3, 4, 5, 6, 7, 8, 9} Se não houver um padrão de repetição o número é irracional e portanto, não poderá ser escrito a/b. Caso o número tenha uma parte não periódica com x dígitos e uma periódica com y dígitos. eseja w = (a_1)/10 + (a_2)/100 +

[obm-l] Re: [obm-l] Questão

2017-08-23 Por tôpico Pedro José
Boa noite! O difícil é achar o n. Como o menor inteiro positivo que atende 10^a = 1 mod23 é a=22 E como 10^3 = 11 mod23. Temos que K + 1 = 3 +22*m com m natural então k = 2 + 22*m. e n/2 = [10^(k+1) -11]/23 ==> n=2*[10^(k+1)-11]/23. Portanto as soluções serão (2+ 22*m; 2*[10^(3+22*m)-11]/23;

[obm-l] Re: [obm-l] Re: [obm-l] Problema de função elementar

2017-07-17 Por tôpico Pedro José
Bom dia! Seguindo a linha proposta pelo Anderson. 7/3^6 < 21/2017 < 8/3^6 ==> F(21/2017)= F(7/3^6)=F(8/3^6) F(7/9) = 3/4. F(7/3^6) = F(7/9/3^4)= F(7/9)/2^4= 3/2^6= 3/64. Sds, PJMS Em 17 de julho de 2017 10:48, Pedro José <petroc...@gmail.com> escreveu: > Bom dia! > &g

[obm-l] Re: [obm-l] Re: [obm-l] Problema de função elementar

2017-07-17 Por tôpico Pedro José
Bom dia! Há uma restrição para a função ser crescente. Portanto F(1) é máximo e F(1) = 1, logo não pode ser 87. tem que ser um valor menor ou igual a 1 e maior ou igual a zero. Sds, PJMS Em 15 de julho de 2017 20:54, Matheus Herculano < matheusherculan...@gmail.com> escreveu: > O resultado é

Re: [obm-l] Geometria plana

2017-07-12 Por tôpico Pedro José
Boa tarde! Só faltaram as definições de a e b, a é a medida do segmento BF e b a do segmento CG. Desculpem-me, PJMS Em 12 de julho de 2017 09:08, Pedro José <petroc...@gmail.com> escreveu: > Bom dia! > > Estava indo pelo caminho errado, derivadas parciais. > > x + y =

Re: [obm-l] Geometria plana

2017-07-12 Por tôpico Pedro José
ica clara a desigualdade e o caso particular da igualdade só se dá para a = b. Portanto, a=b é a condição e a área máxima é 1/4. Atentar que a e b não podem assumir os valores 0 ou 1. Saudações, PJMS Em 11 de julho de 2017 20:50, Pedro José <petroc...@gmail.com> escreveu: > Boa noite! > > N

Re: [obm-l] Geometria plana

2017-07-11 Por tôpico Pedro José
Boa noite! Não consegui por completo, mas a solução é 1/4 e vale para BF=CG . BF<>0 e BF <>1 S(PFQG) = S(FCD) - S(QCG) - S(PGD) ==> S(PFQG) = 1/2 - S(QCG) - S(PGD) (i) S (AGD) + S(BCG) = CG/2 +GD/2 = 1/2 S(QCG) + S(PGD) + S(APD) + S(BCQ) = S (AGD) + S(BCG) = 1/2 (ii) por (i), se S(PFQG) é

[obm-l] Re: [obm-l] Re: [obm-l] Re: [obm-l] Teoria dos números

2017-07-07 Por tôpico Pedro José
Em 7 de julho de 2017 10:36, Pedro José <petroc...@gmail.com> escreveu: > Bom dia! > > Faltou um pedacinho. > > 2) a < max(b,c) > > (i) b >= c > > c=2 ==> a^b+b^2=2ab ==> a >b/2 ==> (b/s)^b < 2ab ==> (b/2)^b<2b^2 > > Para b>=3

[obm-l] Re: [obm-l] Re: [obm-l] Re: [obm-l] Teoria dos números

2017-07-07 Por tôpico Pedro José
Bom dia! Desculpe-me pela solução. Não consegui nada elegante, fui para grosseria. Fui fatiando. 1) a >= max(b,c) (i) a=b=c ==> b<=3; pois a^b+b^c> a^b e a^b>abc=a^3 se a>4. Por paridade só 2 atende, testando é solução. (2,2,2) (ii) a=b>c ==> b<=2; pois, a^3 +b^c> a^3>abc=a^2c b=1 absurdo,

[obm-l] Re: [obm-l] Re: [obm-l] Re: [obm-l] Re: [obm-l] Sugestão de material para OBM

2017-07-05 Por tôpico Pedro José
Em 5 de julho de 2017 01:34, Max Alexandre <maxmalexan...@gmail.com> escreveu: > Obrigado pela sugestão, Pedro José. > > Eu estou tentando formar uma base sim, porque estou praticamente começando > do zero. Já peguei umas aulas do curso da UFRGS voltado a preparar alunos > de

[obm-l] Re: [obm-l] Re: [obm-l] Sugestão de material para OBM

2017-07-04 Por tôpico Pedro José
Boa tarde! Desculpe-me, pela intromissão. Mas você, que não é da área de exatas, fica difícil enveredar de cara no nível universitário. Procure começar pelo nível médio. Os problemas já são cascas-grossas. Primeiro se erguer, depois andar e por fim correr, é o que costumo dizer a minha filha.

Re: [obm-l] Re: [obm-l] Geometria plana (Ajuda)

2017-07-03 Por tôpico Pedro José
Boa noite! Bela e simples solução! Saudações, PJMS Em 29 de junho de 2017 18:21, Julio César Saldaña escreveu: > > > Aproveitando que APC é isósceles (pois CA=CP), eu desenhei a altura CH, > então > AH=HP e anguloACH=anguloHCP=20; mas como também anguloPCB=20, decidi >

Re: [obm-l] Geometria plana (Ajuda)

2017-06-28 Por tôpico Pedro José
Bom dia! O ponto F não foi definido, mas foram definidas duas medidas de ângulos aos quais o ponto F pertence: BCF=20 graus e FCA=40 graus. Não faltou definir o ponto F? Sds, PJMS Em 28 de junho de 2017 09:15, Douglas Oliveira de Lima < profdouglaso.del...@gmail.com> escreveu: > Olá meus

[obm-l] Re: [obm-l] Teoria dos números

2017-06-22 Por tôpico Pedro José
Boa noite! Desculpem-me, faltou o número da revista é a 38. Saudações, PJMS Em 22 de junho de 2017 16:50, Pedro José <petroc...@gmail.com> escreveu: > Boa tarde! > > Esse problema é da OBM 2012, tem uma solução na revista Eureka, página 59, > sitío:http: //www.

[obm-l] Re: [obm-l] Teoria dos números

2017-06-22 Por tôpico Pedro José
Boa tarde! Esse problema é da OBM 2012, tem uma solução na revista Eureka, página 59, sitío:http: //www.obm.org.br/opencms/revista_eureka/ É uma solução longa, bem trabalhosa. O número é 71. E precisa conhecer resíduos quadráticos. Saudações, PJMS Em 22 de junho de 2017 09:47, vinicius

Re: [obm-l] Sistema.

2017-06-06 Por tôpico Pedro José
Boa tarde! É um problema chatinho, embora a resposta seja interessante. O sistema apresentado é indeterminado, não obstante x ser constante. (i) a/b + c/d = -1 (ii)a^2 + c^2 = 1 (iii) b^2 + d^2 = 1 x = b^3/a + d^3/c de (i) a/b = -1 - c/ d ==> (iv) b/a = - d/(c+d) de (i) c/d = -1 -

Re: [obm-l] Problema da olimpiada hungara.

2017-05-22 Por tôpico Pedro José
Boa noite. Tentei da última vez escrever de uma forma simples, mas não deu, tem muitas falhas, não vale, Na verdade, vai se formar um período a partir da anomalia do algarismo das dezenas que é 1 e é a única vez que ele aparece. Depois será formado um período 023456789, que irá valer a

Re: [obm-l] Problema da olimpiada hungara.

2017-05-17 Por tôpico Pedro José
Bom dia! Minha dúvida é de interpretação do português e não quanto a matemática. Quando se fala septuagésima terceira posição a partir do algarismo das unidades, fica dúvida inclusive ou exclusive? É mais fácil perguntar o algarismo de ordem 10^a. pois, dessa forma ficaria claro. Vou supor que é

Re: [obm-l] desigualdade

2017-05-09 Por tôpico Pedro José
Não acerto uma, e z/(z+x)<1 (não z/(z+x)<0,5) ==> x/(x+y) + y/ (y+z) + z/(z+x) < 2. Em 8 de maio de 2017 10:16, Pedro José <petroc...@gmail.com> escreveu: > Bom dia! > > sendo x< y < z, a afirmação que fizera é errônea: o que dará a maior soma > é x , y = x

Re: [obm-l] desigualdade

2017-05-08 Por tôpico Pedro José
> > Em 7 de maio de 2017 23:58, Anderson Torres <torres.anderson...@gmail.com> > escreveu: > >> x/(x+y) + y/ (y+z) + z/(z+x) >> >> 1/(1+y/x) + 1/ (1+z/y) + 1/(1+x/z) >> >> 1/(1+A) + 1/ (1+B) + 1/(1+C) com ABC=1 >> >> talvez dê para pros

Re: [obm-l] desigualdade

2017-05-02 Por tôpico Pedro José
Se pelo menos dois números forem iguais é fácil mostrar que a soma dará 1,5 <= 2. Para x, y e z diferentes, vamos supor x < y x/(2x+1) + y/(2y+1

[obm-l] Re: [obm-l] Re: [obm-l] Re: [obm-l] Probabilidade e números primos

2017-04-10 Por tôpico Pedro José
Desculpem-me, Li tudo errado.p^2 é quem divide. Em 10 de abril de 2017 10:22, Pedro José <petroc...@gmail.com> escreveu: > Bom dia! > > Essa aí eu boiei. > > Os únicos números que dividem p^2 são 1, p e p^2. Serão sempre 3 divisores. > > O universo de n, deveria s

[obm-l] Re: [obm-l] Re: [obm-l] Re: [obm-l] Probabilidade e números primos

2017-04-10 Por tôpico Pedro José
Bom dia! Essa aí eu boiei. Os únicos números que dividem p^2 são 1, p e p^2. Serão sempre 3 divisores. O universo de n, deveria ser limitado a 3*p^2 números, sempre, não faz muito sentido. Não entendi o problema. Saudações, PJFMS. Em 8 de abril de 2017 08:48, Israel Meireles Chrisostomo <

Re: [obm-l] Probleminha bacana

2017-03-13 Por tôpico Pedro José
a. Mas você tem razão, rigorosamente o enunciado precisaria ser >>> melhor, aliás, ser posto de uma forma correta. Mas acredito fortemente que >>> era isso que se passava na cabeça de que elaborou. >>> >>> Em 3 de março de 2017 22:10, Pedro José <petroc...@g

[obm-l] Re: [obm-l] Re: [obm-l] Re: [obm-l] Re: [obm-l] Re: [obm-l] Estou tentando e não sai

2017-03-13 Por tôpico Pedro José
ual a 1 então , S=p , assim > 1=k(mn-k^2), logo só teremos k=1 e mn=2. Assim o único triângulo será o > 3,4,5. > > Abraços > Douglas Oliveira. > > > Em 8 de mar de 2017 8:22 PM, "Pedro José" <petroc...@gmail.com> escreveu: > >> Boa tarde! >> >

[obm-l] Re: [obm-l] Re: [obm-l] Re: [obm-l] Estou tentando e não sai

2017-03-08 Por tôpico Pedro José
> Escreve A = (-a+b+c), B = (a-b+c), C = (a+b-c), assim A+B+C=a+b+c, e > > ABC = 4 (A+B+C) > > Isso dá para ir limitando com desigualdades e recorrer a tentativa e erro. > > 1/4 = 1/(AB) + 1/(AC) + 1/(BC) > > Em 6 de março de 2017 20:08, Pedro José <petroc...@gmail.com&

[obm-l] Re: [obm-l] Estou tentando e não sai

2017-03-06 Por tôpico Pedro José
permutações e não combinações. Em 6 de março de 2017 20:06, Pedro José <petroc...@gmail.com> escreveu: > > Boa noite! > > Fui por aí e achei: > > 4(a+b+c) = a^2b+a^2c+ab^2+ac^2+b^2c+bc^2-2abc-a^3-b^3-c^3 > > Se for triângulo equilátero. > > a=b=c ==>

[obm-l] Re: [obm-l] Estou tentando e não sai

2017-03-06 Por tôpico Pedro José
Boa noite! Fui por aí e achei: 4(a+b+c) = a^2b+a^2c+ab^2+ac^2+b^2c+bc^2-2abc-a^3-b^3-c^3 Se for triângulo equilátero. a=b=c ==> 12a = a^3 ==> a=b=c=raiz(12), que não é inteiro. Se for isósceles com a<>b=c, sem perda de generalidade, pois a equação é simétrica em a,b,c. a^3 -2ba^2+4a + 8b =0

Re: [obm-l] Probleminha bacana

2017-03-03 Por tôpico Pedro José
Boa noite! Não compreendi o problema. Para mim há uma curva de distribuição de probabilidade. Portanto não há como aplicar conceito de modelo discreto. Mas sim integral. Também, não entendi o que significa probabilidade uniforme. Saudações, PJMS Em 3 de março de 2017 11:45, Carlos Gomes

[obm-l] Re: [obm-l] número racional

2017-03-03 Por tôpico Pedro José
Boa noite! Retificação: Portanto só sobram k=2 ou k =22 e não "Portanto só sobram k=2 ou k =11." k é par. Saudações, PJMS Em 2 de março de 2017 09:45, Pedro José <petroc...@gmail.com> escreveu: > Bom dia! > > 4n-2 = k*a^2 (i) e n+5 = K*b^2. > > de (i) tem

[obm-l] Re: [obm-l] Re: [obm-l] Re: [obm-l] Cálculo de determinante.

2017-03-02 Por tôpico Pedro José
Bom dia! A proposição está no Eureka 9, problemas propostos, problema 50, página 59. A solução está na revista seguinte, Eureka10, página 54. Saudações, PJMS Em 28 de fevereiro de 2017 22:10, Douglas Oliveira de Lima < profdouglaso.del...@gmail.com> escreveu: > Realmente não da uma potência de

[obm-l] Re: [obm-l] número racional

2017-03-02 Por tôpico Pedro José
Bom dia! 4n-2 = k*a^2 (i) e n+5 = K*b^2. de (i) temos que *a* pertence a 2 Z+1 e k pertence a 2Z. n = (k*a^2 + 2)/ 4 e n = K*b^2 -5 ==> k (a^2 - (2b)^2) = -22 k=-2 ==> n <=0 e k= -22 ==> n< 0. Portanto só sobram k=2 ou k =11. k=2 ==> (a+2b)*(a-2b)= -11 a+2b=1 e a-2b =-11; a+2b =-1 e a+2b

[obm-l] Re: [obm-l] Intervalos de crescimento de uma função.

2017-02-22 Por tôpico Pedro José
Bom dia! E a função e seu domínio?? Saudações, PJMS Em 22 de fevereiro de 2017 07:14, Douglas Oliveira de Lima < profdouglaso.del...@gmail.com> escreveu: > Olá caros amigos, tenho uma dúvida com relaçao ao intervalo de crescimento > de uma função. > Peguei uma questão da prova da UFF RJ

[obm-l] Re: [obm-l] Re: [obm-l] Conguência

2017-02-15 Por tôpico Pedro José
Boa tarde! Pegando carona na resolução do Marcone. k = 1..11 = 1/9 * (10^81-1) = 1/9 ( (9+1)^81 - 1) = 1/9 (9^81 + 81. 9^80 + ...+ 81*9) = 0 mod81. Saudações. Em 13 de fevereiro de 2017 12:49, Pacini Bores escreveu: > > > > Olá Marcone, > > será que a ideia a

[obm-l] Re: [obm-l] Re: [obm-l] Conguência

2017-02-13 Por tôpico Pedro José
.. + 1*10^7 + 2*10^6 + 3*10^5 + 4*10^4 + 5* 10^3 + 6*10^2 + 7*10 +9 = k*37 = 0 mod9 ==> k= 9a; a>o e a natural. 81| ...111 com 81a algarismos, com a natural e a> 0. Saudações, PJMS Em 13 de fevereiro de 2017 10:48, Pedro José <petroc...@gmail.com> escreveu: > Bom dia!

[obm-l] Re: [obm-l] Re: [obm-l] Conguência

2017-02-13 Por tôpico Pedro José
Bom dia! x = 1...111 (81 algarismos) x= (10^81-1)/9 81 | x ==> 3^6 | 10^81 -1 ==> 10^81 = 1 mod 3^6 a = 1 mod 3^6 ==> a = 1 mod 3^3. Achando ord2710, ou seja, o menor natural d <> 0 onde 10^d = 1 mod 27. Como ord2710 | φ(27)=18; possíveis candidatos: 1, 2, 3, 6,9 , 18. 1 não; 2 não e 3

[obm-l] Re: [obm-l] Re: [obm-l] Re: [obm-l] Sistema de equações

2017-02-07 Por tôpico Pedro José
Boa tarde! Perdão. Faltou uma restrição. C1+C2= 2AB/3 - 4A^3/27. Saudações. Em 7 de fevereiro de 2017 11:20, Pedro José <petroc...@gmail.com> escreveu: > Bom dia! > > A curiosidade estendida: > > Sejam os polinômios P1(x) = x^3 + Ax^2 + Bx + C1 e P2(x) = x^3 + Ax^2 + Bx

[obm-l] Re: [obm-l] Re: [obm-l] Re: [obm-l] Sistema de equações

2017-02-07 Por tôpico Pedro José
Bom dia! A curiosidade estendida: Sejam os polinômios P1(x) = x^3 + Ax^2 + Bx + C1 e P2(x) = x^3 + Ax^2 + Bx + C2 com A, B, C1 e C2 reais e 4A^2<12B. A soma das raízes reais dos polinômios dará - 2A/3. Saudações Em 6 de fevereiro de 2017 20:36, Pedro José <petroc...@gmail.com>

[obm-l] Re: [obm-l] Re: [obm-l] Re: [obm-l] Sistema de equações

2017-02-06 Por tôpico Pedro José
Boa noite! Curiosidade: se os polinômios forem x^3 - 3x^2 +5x + c1 e y^3 - 3y^2 + 5y +c2 e c1 +c2 = -6, a soma das raízes reais do polinômio dará 2. Saudações. Em 6 de fevereiro de 2017 16:37, Pedro José <petroc...@gmail.com> escreveu: > Boa tarde! > > Bela solução. >

[obm-l] Re: [obm-l] Re: [obm-l] Re: [obm-l] Sistema de equações

2017-02-06 Por tôpico Pedro José
Boa tarde! Bela solução. Já eu, fui para a grosseria. Achei as raízes reais das duas equações. x= (-1+ (35/27)^1/2)^1/3 + (-1 - (35/27)^1/2)^1/3 + 1 y = (1 + (35/27)^1/2)^1/3 + (1 -(35/27)^1/2)^1/3 + 1 x+ y =2. Não há outras raízes reais, pois ambos polinômios, x^3 -3x^2 + 5x e y^2-3y^2+5y,

Re: [obm-l] Combinatoria

2016-12-26 Por tôpico Pedro José
Boa tarde! Acho que por contagem só é garantido para 32 ou mais. Por isso o mínimo é 32, com a restrição de garantido. Pois usando a técnica proposta será o máximo de vezes que poderá ser tentado. Ele foi muito feliz na escolha das casas dos pombos. Foi sensacional. Enquanto uns, foram

Re: [obm-l] Combinatoria

2016-12-26 Por tôpico Pedro José
Bom dia! Fui inocente, fiz uma restrição que não precisava. Não há necessidade de acaso. Pode haver estratégia. Saudações, PJMS Em 25 de dezembro de 2016 12:31, Matheus Herculano < matheusherculan...@gmail.com> escreveu: > 87 > > Em 23 de dez de 2016 13:07, "Gabriel Tostes"

Re: [obm-l] Combinatoria

2016-12-23 Por tôpico Pedro José
Bom dia! Novamente o problema está mal formulado. Embora possa parecer claro, qual é o número mínimo de tentativas que *garanta *abrir o armário*.* Dois casos disjuntos atendem. (i) Dois cadeados corretos e o outro errado. Há uma chance de cada cadeado estar correto e 7 chances do terceiro

Re: [obm-l] Teoria dos numeros

2016-12-22 Por tôpico Pedro José
Bom dia! x= 0 y= 1 e z= 1 ; a = -1, b=-1 e c=-1 -1.0 + -1.1 + -1.1 = -1 + 0 -1 (V) atende a 1 + 1 =1 > = 0 +1 +1 (V) atende b. -1 não é soma de três quadrados de inteiros. Tem que ter mais restrições. Saudações, PJMS Em 20 de dezembro de 2016 19:08, Gabriel Tostes

Re: [obm-l] Algebra

2016-12-22 Por tôpico Pedro José
Eerrata: Agora é só pegar os termos que dêm coeficientes >=0, quando multiplicados por 16z^3, ou seja, o expoente de z deverá ser no mínimo -3. Agora é ... expoentes, quando ... Saudações, PJMS. Em 20 de dezembro de 2016 17:28, Pedro José <petroc...@gmail.com> escreveu: > Boa ta

Re: [obm-l] Algebra

2016-12-20 Por tôpico Pedro José
Boa tarde! Ele primeiramente coloca z^6 em evidência em z^6 -z^5+z^4-z^3+z^2-z+1 e obtém z6 (1- 1/z + 1/z^2 - 1/z^3 + 1/z^4 - 1/z^5 + 1/z^6) Como está multiplicado por 16^2, quando aplica a raiz fica : 16 z^3 * raiz (1- 1/z + 1/z^2 - 1/z^3 + 1/z^4 - 1/z^5 + 1/z^6) Aí ele desensenvolve a Série

[obm-l] Re: [obm-l] Re: [obm-l] Princípio da casa dos pombos

2016-12-19 Por tôpico Pedro José
Bom dia! Retificando. (ii)...Portanto, não há como ter mais de um rei *da mesma cor* no tabuleiro, Em 19 de dezembro de 2016 08:15, Pedro José <petroc...@gmail.com> escreveu: > Bom dia! > > Problema complicado. > > (i) Quando se promove um peão não se pode escolher um

[obm-l] Re: [obm-l] Re: [obm-l] Princípio da casa dos pombos

2016-12-19 Por tôpico Pedro José
Bom dia! Problema complicado. (i) Quando se promove um peão não se pode escolher um rei. Portanto não há como ter mais de um rei no tabuleiro. (ii) Um rei não pode estar em cheque por outro rei, é uma jogada impossível. O problema fere dois preceitos básicos do jogo de xadrez. Se esquecermos

[obm-l] Re: [obm-l] Soma de duas frações irredutíveis

2016-11-25 Por tôpico Pedro José
Boa tarde! a/b + c/d e (a,b)=1 e (c,d)=1 a/b + c/d = (ad+bc)/bd Se a/b + c/d é inteiro ==> bd | (ad + bc) ==> b|d e d|b b| d <=. |b| <= |d| d | b ==> |d| <= |b| Então temos que |b| = |d|. Portanto, creio que deva ser inserida mais uma restrição no problema. soma de duas frações

[obm-l] Re: [obm-l] Re: [obm-l] Re: [obm-l] Re: [obm-l] Re: [obm-l] Re: [obm-l] Polinômio irredutível em Z

2016-11-24 Por tôpico Pedro José
Boa noite! Com a observação do Gugu, ficou fácil compreender a filosofia da solução; pois, antes eu estava assim: "Marte chamando Terra, responda!". O contra exemplo apresentado pelo Anderson Torres, não atende o fato de cada par de coeficientes do polinômios terem o mdc =1, como proposto. Porém,

Re: [obm-l] Fatorial

2016-11-17 Por tôpico Pedro José
escreveu: > Obrigado Pedro José > > Em 16 de novembro de 2016 10:29, Pedro José <petroc...@gmail.com> > escreveu: > >> Bom dia! >> >> O fato de haver um múltiplo para cada fator do fatorial não garante a >> divisibilidade, posto que os múltiplos não são

Re: [obm-l] Primos - uma luz

2016-11-16 Por tôpico Pedro José
16 de novembro de 2016 14:34, Pedro José <petroc...@gmail.com> escreveu: > Meu computador está louco. > novo envio espúrio > a=1 e b=3 atende pois 5 = (7*17+1)/24. > > Não foi resolvido. > > Saudações, > PJMS > > Em 16 de novembro de 2016 14:32, Pedro J

Re: [obm-l] Primos - uma luz

2016-11-16 Por tôpico Pedro José
Meu computador está louco. novo envio espúrio a=1 e b=3 atende pois 5 = (7*17+1)/24. Não foi resolvido. Saudações, PJMS Em 16 de novembro de 2016 14:32, Pedro José <petroc...@gmail.com> escreveu: > envio espúrio. > > a=1 e q=3 atende. > > Em 16 de novembro de 2016 14:3

Re: [obm-l] Primos - uma luz

2016-11-16 Por tôpico Pedro José
(a,b) <> (1,3) e (a,b) <>(3,1). a=0 ==> p=2 e q= -3, absurdo, pois -3 não é primo. a=b=1 ==> 5= 49+1/14, absurdo a=1 e b=2 ==> q= 12, não é primo. Absurdo a=1 e q=3 ==> Em 16 de novembro de 2016 13:59, Pedro José <petroc...@gmail.com> escreveu: > Bom dia! > &

Re: [obm-l] Primos - uma luz

2016-11-16 Por tôpico Pedro José
envio espúrio. a=1 e q=3 atende. Em 16 de novembro de 2016 14:31, Pedro José <petroc...@gmail.com> escreveu: > Boa tarde! > > Ficou capenga, pois desse jeito, faltou (1,x), para 5ab > 5 (a+b), e o > operador lógico seria e e não ou. > > Porém mudando a igualdade

Re: [obm-l] Primos - uma luz

2016-11-16 Por tôpico Pedro José
Bom dia! r=2 e p=3 e q = 5 atende. r=3 e p=5 e q = 7 atende r=5 ==> pq = 4 mod5 Já que a solução em p e q é simétrica, analisaremos a impossibilidade só do conjunto de pares (pi,qi) em que (qi,pi) não pertença a esse conjunto, salvo pi=qi. p= 1 mod5 e q = 4 mod5, absurdo; pois p =1 (não é

Re: [obm-l] Fatorial

2016-11-16 Por tôpico Pedro José
Bom dia! O fato de haver um múltiplo para cada fator do fatorial não garante a divisibilidade, posto que os múltiplos não são necessariamente diferentes e nem todos os pares de fatores tem mdc igual a 1. Se zero fizer parte da sequência, está provado. pois n! | 0 para todo n. Veremos agora as

Re: [obm-l] Re: Problema de geometria.

2016-11-03 Por tôpico Pedro José
Boa tarde! Favor postar a solução. Até agora, só rodando em círculos. Em 3 de novembro de 2016 14:53, Douglas Oliveira de Lima < profdouglaso.del...@gmail.com> escreveu: > Opa Carlos , ainda pensei em te ligar rsrsrs, mas eu achei essa raiz ai > sim, na equação do terceiro grau, > fiquei com

[obm-l] Re: [obm-l] Re: [obm-l] Divisibilidade Simultânea (Reformulada)

2016-10-24 Por tôpico Pedro José
Bom dia! Israel, é n+1 | m^2 + 1 e m+1 | n^2 + 1 e não o contrário. Esse problema parece carne de pescoço. Saudações, PJMS. Em 22 de outubro de 2016 13:54, Israel Meireles Chrisostomo < israelmchrisost...@gmail.com> escreveu: > Opa desculpa errei de novo, mas talvez esse seja um caminho > >

[obm-l] (n,m) é múltiplo de n.

2016-10-17 Por tôpico Pedro José
Bom dia! Revisando a solução anterior. 1) Se mdc (n,m)= 1 então (n,m) é múltiplo de n. Pois não existirá um primo que divida n e (n-m), que veremos a seguir que é condicionante para que não seja múltiplo. E engloba casos triviais como (n,1) e (n,n-1). Nota: o item 2 é suficiente para

<    1   2   3   4   5   6   >